Multivariable Calculus, 7th Edition

Published by Brooks Cole
ISBN 10: 0-53849-787-4
ISBN 13: 978-0-53849-787-9

Chapter 11 - Infinite Sequences and Series - 11.3 Exercises - Page 745: 16

Answer

Divergent

Work Step by Step

Given $$ \sum_{n=1}^{\infty} \frac{n^{2}}{n^{3}+1}$$ Since $f(x)=\dfrac{x^{2}}{x^{3}+1}$ is continuous and positive on $[2, \infty)$ and $$ f^{\prime}(x)=\frac{x\left(2-x^{3}\right)}{\left(x^{3}+1\right)^{2}}<0$$ for all $x\geq 2 $, then by using the Integral Test, we get \begin{align*} \int_{2}^{\infty} \frac{x^{2}}{x^{3}+1} d x&=\frac{1}{3}\lim _{t \rightarrow \infty}\left( \ln \left(x^{3}+1\right)\right)\bigg|_{2}^{t}\\ &=\frac{1}{3} \lim _{t \rightarrow \infty}\left(\ln \left(t^{3}+1\right)-\ln 9\right)\\ &=\infty \end{align*} Hence $\displaystyle\sum_{n=2}^{\infty} \frac{n^{2}}{n^{3}+1}$ diverges and so does the given series $\displaystyle\sum_{n=1}^{\infty} \frac{n^{2}}{n^{3}+1}$
Update this answer!

You can help us out by revising, improving and updating this answer.

Update this answer

After you claim an answer you’ll have 24 hours to send in a draft. An editor will review the submission and either publish your submission or provide feedback.